2. Trigonometric Substitutions

Exercises

    Evaluate each integral.

  1. \( \displaystyle \int \dfrac{\sqrt{x^{2}-1}}{x}\,dx \)

    \(\sec(u)=x\)

    \( \displaystyle \int \dfrac{\sqrt{x^{2}-1}}{x}\,dx=\sqrt{x^{2}-1}-\text{arcsec}x+C \)

    Use the trigonometric substitution \(x=\sec(u)\). Then \(dx=\tan(u)\sec(u)\,du\). \[\begin{aligned} \int\dfrac{\sqrt{x^2-1}}{x}\,dx&=\int\dfrac{\tan(u)}{\sec(u)}\tan(u)\sec(u)\,du \\ &=\int\tan^2(u)\,du \\ &=\int\sec^2(u)-1\,du \\ &=\tan(u)-u+C \\ &=\sqrt{x^2-1}-\text{arcsec}(x)+C \end{aligned}\]

    pm 

    To check, we take the derivative. \[\begin{aligned} \left(\sqrt{x^2-1}-\text{arcsec}(x)\right)'&=2x\dfrac{1}{2\sqrt{x^2-1}}-\dfrac{1}{x\sqrt{x^2-1}} \\ &=\dfrac{x^2}{x\sqrt{x^2-1}}\,-\,\dfrac{1}{x\sqrt{x^2-1}} \\ &=\dfrac{x^2-1}{x\sqrt{x^2-1}} \\ &=\dfrac{\sqrt{x^2-1}}{x} \end{aligned}\]

  2. \(\displaystyle\int\dfrac{x}{\left(x^2+1\right)^{3/2}}\,dx\)

    Use the ordinary substitution \(u=x^2+1\).

    \(\displaystyle\int\dfrac{x}{\left(x^2+1\right)^{3/2}}\,dx =-\dfrac{1}{\sqrt{x^2+1}}+C\)

    Use the ordinary substitution \(u=x^2+1\). Then \(du=2x\,dx\) or \(\dfrac{1}{2}\,du=x\,dx\). So: \[\begin{aligned} \int \dfrac{x}{(x^2+1)^{3/2}}\,dx &=\dfrac{1}{2}\int u^{-3/2}\,du \\ &=\dfrac{1}{2}(-2)u^{-1/2}+C =-\,\dfrac{1}{\sqrt{u}}+C \\ &=-\,\dfrac{1}{\sqrt{x^2+1}}+C \end{aligned}\]

    vm,pm 

    We check by differentiating. If \(f(x)=-\dfrac{1}{\sqrt{x^2+1}}\) then: \[ f'(x)=\dfrac{1}{2}\dfrac{1}{(x^2+1)^{3/2}}(2x) =\dfrac{x}{(x^2+1)^{3/2}} \] which is the original integrand.

  3. \( \displaystyle \int \dfrac{x}{\sqrt{x^{2}+1}}\,dx \)

    Use the simpler method, \(u\)-substitution...

    \( \displaystyle \int \dfrac{x}{\sqrt{x^{2}+1}}\,dx=\sqrt{x^{2}+1}+C \)

    We use the ordinary substitution \(u=x^2+1\). Then \(du=2x\,dx\) or \(\dfrac{1}{2}\,du=x\,dx\). So: \[\begin{aligned} \int \dfrac{x}{\sqrt{x^{2}+1}}\,dx&=\dfrac{1}{2}\int u^{-1/2}\,du \\ &=\dfrac{1}{2}(2)u^{1/2}+C \\ &=\dfrac{1}{\sqrt{u}}+C \\ &=\sqrt{x^2+1}+C \\ \end{aligned}\]

    pm 

    Check by differentiating. \[\begin{aligned} (\sqrt{x^2+1})'&=\dfrac{1}{2\sqrt{x^2+1}}(x^2+1)' \\ &=\dfrac{2x}{2\sqrt{x^2+1}} \\ &=\dfrac{x}{\sqrt{x^2+1}} \end{aligned}\]

  4. \( \displaystyle \int \dfrac{5x-4}{\sqrt{1-x^{2}}}\,dx \)

    Use the substitution \(x=\sin(\theta)\).

    \( \displaystyle \int \dfrac{5x-4}{\sqrt{1-x^{2}}}\,dx=-5\sqrt{1-x^{2}}-4\arcsin x+C\)

    We use the trigonometric substitution \(x=\sin(u)\). Then \(dx=\cos(u)\,du\). So: \[\begin{aligned} \displaystyle \int \dfrac{5x-4}{\sqrt{1-x^{2}}}\,dx &= \displaystyle \int \dfrac{5\sin(u)-4}{\sqrt{1-\sin^2u}}\cos(u)\,du \\ &= \displaystyle \int (5\sin(u)-4)\,du \\ &= -5\cos(u)-4u+C \\ &= -5\sqrt{1-x^2}-4\arcsin(x)+C \\ \end{aligned}\]

    pm 

    Check by differentiating. If \(f=-5\sqrt{1-x^2}-4\arcsin(x)\), then \[\begin{aligned} f'&=-5\dfrac{(1-x^2)'}{2\sqrt{1-x^2}}-4\dfrac{1}{\sqrt{1-x^2}} \\ &=-5\dfrac{-2x}{2\sqrt{1-x^2}}-\dfrac{4}{\sqrt{1-x^2}} \\ &=\dfrac{5x-4}{\sqrt{1-x^2}} \end{aligned}\]


  5. Verify the antiderivative formula: \(\displaystyle \int \dfrac{\sqrt{1-x^{2} }}{x^{4}}\,dx=-\,\dfrac{(1-x^2)^{3/2}}{3x^3}+C \)

    Remember to verify an antiderivative requires only a derivative!

    To verify the antiderivative, we let \(f=-\,\dfrac{(1-x^2)^{3/2}}{3x^3}\) and differentiate: \[\begin{aligned} f'&=-\,\dfrac{(3x^3)[(1-x^2)^{3/2}]'-(1-x^2)^{3/2}[3x^3]'}{(3x^3)^2} \\ &=-\dfrac{3x^3\dfrac{3}{2}(1-x^2)^{1/2}(-2x)-(1-x^2)^{3/2}9x^2}{9x^6} \\ &=-\,\dfrac{-9x^4\sqrt{1-x^2}-9x^2(1-x^2)\sqrt{1-x^2}}{9x^6} \\ &=-\,\dfrac{(-9x^4-9x^2+9x^4)\sqrt{1-x^2}}{9x^6} \\ &=\dfrac{\sqrt{1-x^2}}{x^4} \end{aligned}\] which is the integrand.

    pm 


  6. Plot the parametric curve \((x,y)=\left(\theta\cos \theta,\theta\sin \theta\right)\) from \(\theta=\dfrac{-3\pi}{2}\) to \(\theta=\dfrac{3\pi}{2}\) and compute its arc length.

    The formula for the arc length of a polar curve is given in the chapter on Polar Coordinates . It is \[ L=\int_\alpha^\beta \sqrt{r(\theta)^2+\left(\dfrac{dr}{d\theta}\right)^2}\,d\theta \] Since \(\theta\) is the variable of integration, you cannot use \(\theta\) as the new variable in the trig substitution.

    Happy Valentine's Day!
    \(\begin{aligned} L&=\dfrac{3\pi}{4}\sqrt{9\pi^2+4} +\dfrac{1}{2}\ln\dfrac{\sqrt{9\pi^2+4}+3\pi}{\sqrt{9\pi^2+4}-3\pi} \\ &\approx24.96 \end{aligned}\)

    x_valentine

    Comparing the curve \((x,y)=\left(\theta\cos \theta,\theta\sin \theta\right)\) to polar coordinates \((x,y)=\left(r\cos \theta,r\sin \theta\right)\), we see this is the polar curve \(r=\theta\). So the radius gets larger as the angle gets larger. It is a spiral as shown. Happy Valentine's Day!

    The arc length of a polar curve is given by: \[ L=\int_\alpha^\beta \sqrt{r(\theta)^2+\left(\dfrac{dr}{d\theta}\right)^2}\,d\theta \] Here \(r=\theta\) and \(\dfrac{dr}{d\theta}=1\) and the limits are \(\alpha=-\,\dfrac{3\pi}{2}\) and \(\beta=\dfrac{3\pi}{2}\). So \[ L=\int_{-3\pi/2}^{3\pi/2} \sqrt{\theta^2+1}\,d\theta \]

    x_valentine

    We make the trig substitution \(\theta=\tan(u)\). Then \(\dfrac{d\theta}{du}=\sec^2(u)\). So \[\begin{aligned} L&=\int_{\theta=-3\pi/2}^{3\pi/2} \sqrt{\tan^2 u+1}\sec^2(u)\,du \\ &=\int_{\theta=-3\pi/2}^{3\pi/2} \sec^3(u)\,du \\ \end{aligned}\] This indefinite integral was computed in the chapter on Trig Integrals. \[ L=\left[\dfrac{1}{2}\sec(u)\tan(u) +\dfrac{1}{2}\ln|\sec(u)+\tan(u)|\right]_{\theta=-3\pi/2}^{3\pi/2} \] To substitute back, we look at a triangle with angle \(u\), opposite side \(\theta\), adjacent side \(1\) and hypotenuse \(\sqrt{\theta^2+1}\): \[\begin{aligned} L&=\left[\dfrac{1}{2}\sqrt{\theta^2+1}\,\theta +\dfrac{1}{2}\ln|\sqrt{\theta^2+1}+\theta|\right]_{-3\pi/2}^{3\pi/2} \\ &=\dfrac{1}{2}\sqrt{\left(\dfrac{3\pi}{2}\right)^2+1}\,\dfrac{3\pi}{2} +\dfrac{1}{2}\ln\left|\sqrt{\left(\dfrac{3\pi}{2}\right)^2+1}+\dfrac{3\pi}{2}\right| \\ &\quad +\dfrac{1}{2}\sqrt{\left(\dfrac{3\pi}{2}\right)^2+1}\,\dfrac{3\pi}{2} -\dfrac{1}{2}\ln\left|\sqrt{\left(\dfrac{3\pi}{2}\right)^2+1}-\dfrac{3\pi}{2}\right| \\ &=\sqrt{\left(\dfrac{3\pi}{2}\right)^2+1}\,\dfrac{3\pi}{2} +\dfrac{1}{2}\ln\dfrac{\sqrt{\left(\dfrac{3\pi}{2}\right)^2+1}+\dfrac{3\pi}{2}} {\sqrt{\left(\dfrac{3\pi}{2}\right)^2+1}-\dfrac{3\pi}{2}} \\ &=\dfrac{\sqrt{9\pi^2+4}}{2}\,\dfrac{3\pi}{2} +\dfrac{1}{2}\ln\dfrac{\dfrac{\sqrt{9\pi^2+4}}{2}+\dfrac{3\pi}{2}} {\dfrac{\sqrt{9\pi^2+4}}{2}-\dfrac{3\pi}{2}} \\ &=\dfrac{3\pi}{4}\sqrt{9\pi^2+4} +\dfrac{1}{2}\ln\dfrac{\sqrt{9\pi^2+4}+3\pi}{\sqrt{9\pi^2+4}-3\pi} \\ &\approx 24.96 \end{aligned}\]

    pm 

  7. Evaluate each integral.

  8. \( \displaystyle \int \dfrac{dx}{\sqrt{x^{2}-4}}\)

    Let \( x=2 \sec \theta \).

    \( \displaystyle \int \dfrac{dx}{\sqrt{x^{2}-4}} =\ln \left\vert \dfrac{x}{2}+\dfrac{\sqrt{x^{2}-4}}{2}\right\vert+K =\ln\left|x+\sqrt{x^2-4}\right|+C \)

    We let \(x=2\sec\theta\). So \(dx=2\sec\theta\tan\theta\,d\theta\). Then \[\begin{aligned} \int\dfrac{dx}{\sqrt{x^{2}-4}} &=\int\dfrac{2\sec\theta\tan\theta\,d\theta}{\sqrt{4\sec^2\theta-4}} =\int\dfrac{2\sec\theta\tan\theta\,d\theta}{\sqrt{4\tan^2\theta}} \\ &=\int\sec\theta\,d\theta=\ln|\sec\theta+\tan\theta|+K \end{aligned}\] Now we must express our result in terms of \(x\). Since \(\sec\theta=\dfrac{x}{2}\), we have: \[ \tan\theta=\sqrt{\sec^2\theta-1}=\sqrt{\dfrac{x^2}{4}-1} =\dfrac{\sqrt{x^2-4}}{2} \] Making these substitutions gives \[\begin{aligned} \int\dfrac{dx}{\sqrt{x^{2}-4}} &=\ln\left|\dfrac{x}{2}+\dfrac{\sqrt{x^2-4}}{2}\right|+K \\ &=\ln\left|\dfrac{x+\sqrt{x^2-4}}{2}\right|+K \\ &=\ln\left|x+\sqrt{x^2-4}\right|-\ln(2)+K \\ &=\ln\left|x+\sqrt{x^2-4}\right|+C \end{aligned}\]

    pm 

    We check by differentiating: \[\begin{aligned} \dfrac{d}{dx}&\left(\ln\left|x+\sqrt{x^2-4}\right|\right) =\dfrac{\dfrac{d}{dx\rule[-2pt]{0pt}{10pt}}\left(x+\sqrt{x^2-4}\right)}{x+\sqrt{x^2-4}} \\ &=\dfrac{1+\dfrac{1}{2}\left[(x^2-4)^{-1/2}\right]2x}{x+\sqrt{x^2-4}} \\ &=\dfrac{1+\dfrac{x}{\sqrt{x^2-4}}}{\;\;x+\sqrt{x^2-4}\;\;} =\dfrac{\dfrac{\sqrt{x^2-4}+x}{\sqrt{x^2-4}}}{\;\;x+\sqrt{x^2-4}\;\;} \\ &=\dfrac{\sqrt{x^2-4}+x}{\sqrt{x^2-4}[x+\sqrt{x^2-4}]} =\dfrac{1}{\sqrt{x^2-4}} \end{aligned}\]

  9. \(\displaystyle \int \dfrac{dx}{\sqrt{9-16x^{2}}}\)

    Let \(x=\dfrac{3}{4}\sin(\theta)\).

    \( \displaystyle \int \dfrac{dx}{\sqrt{9-16x^{2}}} =\dfrac{1}{4}\arcsin\left(\dfrac{4}{3}x\right)+C\)

    Let \(x=\dfrac{3}{4}\sin(\theta)\). Then \(dx=\dfrac{3}{4}\cos(\theta)d\theta\). So: \[\begin{aligned} \int &\dfrac{dx}{\sqrt{9-16x^2}} =\int \dfrac{1}{\sqrt{9-9\sin^2(\theta)}}\dfrac{3}{4}\cos(\theta)d\theta =\int \dfrac{1}{3\cos(\theta)}\dfrac{3}{4}\cos(\theta)d\theta \\ &=\int \dfrac{1}{4} d\theta =\dfrac{\theta}{4}+C =\dfrac{1}{4}\arcsin\left(\dfrac{4}{3}x\right)+C \end{aligned}\]

    pm 

    \[\begin{aligned} \left(\dfrac{1}{4}\arcsin\left(\dfrac{4}{3}x\right)\right)' &=\dfrac{1}{4}\left[\dfrac{4}{3}\dfrac{1}{\sqrt{1-\left(\dfrac{4}{3}x\right)^2}}\right] \\ &=\dfrac{1}{3}\dfrac{1}{\sqrt{\dfrac{9-16x^2}{9}}} \\ &=\dfrac{1}{3}\dfrac{3}{\sqrt{9-16x^2}} \\ &=\dfrac{1}{\sqrt{9-16x^2}} \end{aligned}\]

    An alternate way to do this problem is to make the ordinary substitution \(x=\dfrac{3}{4}u\). Then \(dx=\dfrac{3}{4}\,du\). So: \[\begin{aligned} \int \dfrac{dx}{\sqrt{9-16x^2}} &=\int \dfrac{1}{\sqrt{9-9u^2}}\dfrac{3}{4}\,du \\ &=\dfrac{1}{4}\int \dfrac{1}{\sqrt{1-u^2}}\,du \\ &=\dfrac{1}{4}\arcsin(u)+C \\ &=\dfrac{1}{4}\arcsin\left(\dfrac{4}{3}x\right)+C \end{aligned}\]

  10. \( \displaystyle \int \dfrac{x^{3}}{\sqrt{9x^{2}+49}}\,dx \)

    Do the normal substitution \(u=9x^2+49\).

    \( \displaystyle \int \dfrac{x^{3}}{\sqrt{9x^{2}+49}}\,dx\) \(\quad =\dfrac{1}{243}(9x^{2}+49)^{3/2}-\dfrac{49}{81}\sqrt{9x^{2}+49}+C \)

    Use the ordinary substitution \(u=9x^{2}+49\) so that \(du=18x\,dx\) and \(x^2=\dfrac{u-49}{9}\). Then: \[\begin{aligned} \int\dfrac{x^3}{\sqrt{9x^2+49}}\,dx &=\int x^2\dfrac{1}{\sqrt{9x^2+49}}x\,dx \\ &=\int\dfrac{u-49}{9}\dfrac{1}{\sqrt{u}}\dfrac{du}{18} \\ &=\dfrac{1}{162}\int\left(\sqrt{u}-\dfrac{49}{\sqrt{u}}\right)\,du \\ &=\dfrac{1}{162}\dfrac{2u^{3/2}}{3}-\dfrac{49}{162}2\sqrt{u}+C \\ &=\dfrac{1}{243}\left(9x^{2}+49\right)^{3/2}-\dfrac{49}{81}\sqrt{9x^{2}+49}+C \end{aligned}\]

    vm,pm 

    We check by differentiating. If \(f(x)=\dfrac{1}{243}\left(9x^{2}+49\right)^{3/2}-\dfrac{49}{81}\sqrt{9x^{2}+49}\), then \[\begin{aligned} f'(x)&=\dfrac{1}{243}\dfrac{3}{2}(9x^2+49)^{1/2}[18x] -\dfrac{49}{81}\dfrac{1}{2\sqrt{9x^2+49}}[18x] \\ &=\dfrac{x}{9}\sqrt{9x^2+49}-\dfrac{49x}{9\sqrt{9x^2+49}} \\ &=\dfrac{x(9x^2+49)-49x}{9\sqrt{9x^2+49}} =\dfrac{9x^3+49x-49x}{9\sqrt{9x^2+49}} \\ &=\dfrac{x^3}{\sqrt{9x^2+49}} \end{aligned}\]

    We can also do the integral by a trig substitution. Let \(x=\dfrac{7}{3}\tan(\theta)\), which implies \(dx=\dfrac{7}{3}\sec^2(\theta)\,d\theta\). Then \[\begin{aligned} \int \dfrac{x^3}{\sqrt{9x^2+49}}\,dx &=\int \dfrac{\dfrac{343}{27}\tan^3(\theta)}{\sqrt{49\tan^2\theta+49}}\dfrac{7}{3}\sec^2(\theta)\,d\theta \\ &=\dfrac{343}{81}\int \dfrac{\tan^3(\theta)\sec^2(\theta)}{\sqrt{\sec^2\theta}}\,d\theta \\ &=\dfrac{343}{81}\int \tan^3(\theta)\sec(\theta)\,d\theta \\ &=\dfrac{343}{81}\int (\sec^2(\theta)-1)\tan(\theta)\sec(\theta)\,d\theta \end{aligned}\] Then let \(u=\sec(\theta)\). Then \(du=\sec(\theta)\tan(\theta)d\theta\). \[\begin{aligned} \int \dfrac{x^3}{\sqrt{9x^2+49}}\,dx &=\dfrac{343}{81}\int u^2-1\,du \\ &=\dfrac{343}{81}\left[\dfrac{u^3}{3}-u\right]+C \end{aligned}\] Since \(u=\sec(\theta)=\sqrt{\tan^2{\theta}+1}=\dfrac{\sqrt{9x^2+49}}{7}\), we have: \[\begin{aligned} \int \dfrac{x^3}{\sqrt{9x^2+49}}\,dx &=\dfrac{343}{81}\left[\dfrac{(9x+49)^{3/2}}{3 \cdot 343}-\dfrac{\sqrt{9x^2+49}}{7}\right]+C \\ &=\dfrac{1}{243}\left(9x^{2}+49\right)^{3/2}-\dfrac{49}{81}\sqrt{9x^{2}+49}+C \end{aligned}\]

    pm 

  11. \( \displaystyle \int_{0}^{6}\sqrt{36-x^{2}}\,dx \)

    A trig substitution will work, but recognizing the geometric figure makes the problem especially simple.

    \( \displaystyle \int_{0}^{6}\sqrt{36-x^{2}}\,dx = 9\pi\)

    The graph of \(y=36-x^2\) for \(0 \le x \le 6\) is a quarter circle with radius \(6\). The area of the whole circle is \(\pi \cdot 6^2 = 36\pi\), so a quarter of it would have an area \[ \int_{0}^{6}\sqrt{36-x^{2}}\,dx = A =\dfrac{1}{4}36\pi= 9\pi \]

    pm 

    The integral can also be computed by the trig substitution \(x=6\sin(\theta)\), which means \(dx\,=6\cos(\theta)\,d\theta\). \[\begin{aligned} \displaystyle \int_0^6 \sqrt{36-x^2} \,dx &= \int_0^{\pi/2} \sqrt{36-36\sin^2(\theta)}\,6\cos(\theta)\,d\theta \\ &=36\int_0^{\pi/2} \sqrt{1-\sin^2(\theta)}\cos(\theta)\,d\theta \\ &=36\int_0^{\pi/2} \cos^2(\theta)\,d\theta \\ &=36\int_0^{\pi/2} \dfrac{1+\cos(2\theta)}{2}\,d\theta \\ &=18\int_0^{\pi/2} 1+\cos(2\theta)\,d\theta\\ &=18\left[\theta+\dfrac{\sin2\theta}{2}\right]_0^{\pi/2} =9\pi \end{aligned}\] Notice this solution was harder than the ordinary substitution.

    pm 


  12. Find the arc length of the portion of \(y=x^{2}\) on the interval \(\left[ 0,\dfrac{\sqrt{3}}{2}\right]\)

    Let \(x=\dfrac{1}{2}\tan(\theta)\).

    \( \displaystyle L=\int_{0}^{\sqrt{3}/2} \sqrt{1+4x^{2}}\,dx=\dfrac{1}{2}\sqrt{3}+\dfrac{1}{4} \ln \left( 2+\sqrt{3}\right) \)

    \[\begin{aligned} L&=\int_a^b \sqrt{1+\left(\dfrac{dy}{dx} \right )^2} dx \\ &=\int_{0}^{\sqrt{3}/2}\sqrt{1+4x^{2}}\,dx \end{aligned}\] Let \(x=\dfrac{1}{2}\tan(\theta)\). Then \(dx=\dfrac{1}{2}\sec^2(\theta)\,d\theta\). \[\begin{aligned} L&=\int_{x=0}^{\sqrt{3}/2}\sqrt{1+\tan^2(\theta)}\dfrac{\sec^2(\theta)}{2}\,d\theta \\ &=\dfrac{1}{2}\int_{x=0}^{\sqrt{3}/2}\sec^3(\theta)\,d\theta \\ \end{aligned}\] We found this integral in the chapter on Trig Integrals: \[ L=\dfrac{1}{2}\left[\dfrac{1}{2}\sec(\theta)\tan(\theta) +\dfrac{1}{2}\ln|\sec(\theta)+\tan(\theta)|\right]_{x=0}^{\sqrt{3}/2} \] Since \(\tan(\theta)=2x\), we know \(\sec(\theta)=\sqrt{4x^2+1}\). So: \[\begin{aligned} L&=\dfrac{1}{4}\left[\sqrt{4x^2+1}\,2x+\ln|\sqrt{4x^2+1}+2x|\right]_{x=0}^{\sqrt{3}/2}\\ &=\dfrac{1}{4}\sqrt{3+1}\sqrt{3}+\dfrac{1}{4}\ln|\sqrt{3+1}+\sqrt{3}|\\ &\quad-\,\dfrac{1}{4}\sqrt{1}\,\cdot 0-\dfrac{1}{4}\ln|\sqrt{1}+0|\\ &=\dfrac{1}{2}\sqrt{3}+\dfrac{1}{4}\ln\left(2+\sqrt{3}\right) \end{aligned}\]

    gk,pm 

  13. Complete the squares in each of these. Then integrate using trig substitutions.

  14. \(\displaystyle \int \dfrac{dx}{\sqrt{x^{2}-2x+5}}\)

    Let \(x-1=2\tan{\theta}\).

    \(\displaystyle \int\dfrac{dx}{\sqrt{x^2-2x+5}} =\ln\left|\sqrt{x^2-2x+5}+x-1\right|+C \)

    Complete the square: \[ \int\dfrac{dx}{\sqrt{x^2-2x+5}} =\int\dfrac{dx}{\sqrt{x^2-2x+1+4}} =\int\dfrac{dx}{\sqrt{(x-1)^2+4}} \] Now let \(x-1=2\tan\theta\). Then \(dx=2\sec^2\theta\,d\theta\) and: \[ \sqrt{(x-1)^2+4}=\sqrt{4\tan^2\theta +4} =\sqrt{4\sec^2\theta}=2\sec\theta \] We now rewrite the integral: \[\begin{aligned} \int &\dfrac{dx}{\sqrt{x^2-2x+5}} =\int\dfrac{2\sec^2\theta\,d\theta}{2\sec\theta} \\ &=\int\sec\theta\,d\theta =\ln|\sec\theta+\tan\theta|+C \end{aligned}\] Finally, to obtain our result in terms of \(x\), we could draw a right triangle or use the formulas: \[\begin{aligned} \tan\theta&=\dfrac{x-1}{2} \\ \sec\theta&=\sqrt{\tan^2\theta+1} =\sqrt{\dfrac{(x-1)^2}{4}+1} \\ &=\dfrac{\sqrt{(x-1)^2+4}}{2} \end{aligned}\] Putting all this together, we get \[\begin{aligned} \int\dfrac{dx}{\sqrt{x^2-2x+5}} &=\ln\left|\dfrac{\sqrt{x^2-2x+5}}{2}+\dfrac{x-1}{2}\right| +C_1 \\ &=\ln\left|\sqrt{x^2-2x+5}+x-1\right| +C \end{aligned}\]

    vm,pm 

    \[\begin{aligned} &\left( \ln\left|\sqrt{x^2-2x+5}+x-1\right| \right)' =\dfrac{\dfrac{2x-2}{2\sqrt{x^2-2x+5}}+1}{\sqrt{x^2-2x+5}+x-1} \\ &\quad=\dfrac{x-1+\sqrt{x^2-2x+5}} {\sqrt{x^2-2x+5}\left(\sqrt{x^2-2x+5}+x-1\right)} \\ &\quad=\dfrac{1}{\sqrt{x^2-2x+5}} \end{aligned}\]

  15. \(\displaystyle \int \dfrac{x-1}{\sqrt{x^{2}-4x+3}}\,dx\)

    Let \(x-2=\sec(\theta)\).

    \(\displaystyle \int \dfrac{x-1}{\sqrt{x^{2}-4x+3}}\,dx =\sqrt{x^2-4x+3}+\ln\left|x-2+\sqrt{x^2-4x+3}\right|+C\)

    We first complete the square: \[ \int \dfrac{x-1}{\sqrt{x^{2}-4x+3}}\,dx =\int \dfrac{x-1}{\sqrt{(x-2)^2-1}}\,dx \] Now let \(x-2=\sec\theta\). Then \(dx=\sec{\theta}\tan{\theta}\,d\theta\). So: \[\begin{aligned} \int \dfrac{x-1}{\sqrt{x^{2}-4x+3}}\,dx &=\int \dfrac{\sec{\theta}+2-1}{\tan{\theta}}\sec{\theta}\tan{\theta}\,d\theta \\ &=\int \sec^2{\theta}+\sec{\theta}\,d\theta \\ &=\tan{\theta}+\ln|\sec{\theta}+\tan{\theta}| +C \\ &=\sqrt{\sec^2{\theta}-1}+\ln\left|\sec{\theta}+\sqrt{\sec^2{\theta}-1}\right|+C \\ &=\sqrt{x^2-4x+3}+\ln\left|x-2+\sqrt{x^2-4x+3}\right|+C \end{aligned}\]

    pm 

    \[\begin{aligned} \dfrac{d}{dx}&\left(\sqrt{x^2-4x+3}+\ln\left|x-2+\sqrt{x^2-4x+3}\right|\right) \\ &=\dfrac{2x-4}{2\sqrt{x^2-4x+3}}+\dfrac{1+\dfrac{2x-4}{2\sqrt{x^2-4x+3}}}{\;x-2+\sqrt{x^2-4x+3}\;} \\ &=\dfrac{x-2}{\sqrt{x^2-4x+3}}+\dfrac{\dfrac{x-2+\sqrt{x^2-4x+3}}{\sqrt{x^2-4x+3}}}{\;\;x-2+\sqrt{x^2-4x+3}\;\;} \\ &=\dfrac{x-2}{\sqrt{x^2-4x+3}}+\dfrac{1}{\sqrt{x^2-4x+3}} \\ &=\dfrac{x-1}{\sqrt{x^2-4x+3}}\\ \end{aligned}\]

  16. \(\displaystyle \int \dfrac{x}{\sqrt{5+4x-x^{2}}}\,dx\)

    Complete the square and let \(x-2=3\sin\theta\).

    \(\displaystyle \int \dfrac{x}{\sqrt{5+4x-x^{2}}}\,dx =-\sqrt{5+4x-x^2}+2\arcsin{\dfrac{x-2}{3}}+C\)

    Complete the square: \[ \int \dfrac{x}{\sqrt{5+4x-x^2}}\,dx =\int \dfrac{x}{\sqrt{9-(x^2-4x+4)}}\,dx =\int \dfrac{x}{\sqrt{9-(x-2)^2}}\,dx \] Now let \(x-2=3\sin\theta\). Then \(dx=3\cos\theta\,d\theta\) and: \[ \sqrt{9-(x-2)^2}=\sqrt{9-9\sin^2\theta} =\sqrt{9\cos^2\theta}=3\cos\theta \] Putting it all together, we get: \[\begin{aligned} \int \dfrac{x}{\sqrt{5+4x-x^2}}\,dx &=\int \dfrac{(3\sin\theta+2)}{3\cos\theta}3\cos\theta\,d\theta \\ &=\int 3\sin\theta+2\,d\theta \\ &=-3\cos\theta+2\theta+C \\ \end{aligned}\] Since \(\sin\theta=\dfrac{x-2}{3}\), we have \(\theta=\arcsin\dfrac{x-2}{3}\) and \[ \cos\theta=\sqrt{1-\sin^2\theta} =\sqrt{1-\dfrac{(x-2)^2}{9}} =\dfrac{\sqrt{9-(x-2)^2}}{3} \] Consequently: \[\begin{aligned} \int \dfrac{x}{\sqrt{5+4x-x^2}}\,dx &=-3\,\dfrac{\sqrt{9-(x-2)^2}}{3}+2\arcsin{\dfrac{x-2}{3}}+C \\ &=-\sqrt{5+4x-x^2}+2\arcsin{\dfrac{x-2}{3}}+C \end{aligned}\]

    vm,pm 

    \[\begin{aligned} \dfrac{d}{dx}&\left(-\sqrt{5+4x-x^2}+2\arcsin\left(\dfrac{x-2}{3}\right)\right) \\ &=-\dfrac{4-2x}{2\sqrt{5+4x-x^2}}+2\dfrac{\dfrac{1}{3}}{\sqrt{1-\dfrac{x^2-4x+4}{9}}} \\ &=\dfrac{x-2}{\sqrt{5+4x-x^2}}+\dfrac{2}{\sqrt{5+4x-x^2}} \\ &=\dfrac{x}{\sqrt{5+4x-x^2}} \end{aligned}\]

  17. Compute each of the following integrals using a hyperbolic trig substitution.

  18. \(\displaystyle\int \dfrac{4}{\sqrt{x^2-1}}dx\)

    Use the substitution \(x = \cosh \lambda\).

    \(\displaystyle\int \dfrac{4}{\sqrt{x^2-1}}dx = 4\mathrm{arccosh}\,x + C = 4\ln\left(x+\sqrt{x^2-1}\right)+C\)

    Let \(x = \cosh \lambda \). Thus,\( \,dx = \sinh \lambda\,d\lambda \). \[\begin{aligned} \int \dfrac{4}{\sqrt{x^2-1}}\,dx &= \int \dfrac{4}{\sqrt{\cosh^2\lambda - 1}}\sinh \lambda \,d\lambda \\ &= \int \dfrac{4}{\sqrt{\sinh^2\lambda}}\sinh \lambda \,d\lambda \\ &= \int 4 \,d\lambda \\ &= 4\lambda + C \\ \end{aligned}\] Next, we substitute back for \(x\). Since \(x = \cosh \lambda \), we know \(\lambda = \mathrm{arccosh}\,x \). Therefore, \[ \int \dfrac{4}{\sqrt{x^2-1}}dx = 4\,\mathrm{arccosh}\,x + C \] While this is a fine answer, \(\mathrm{arccosh}\,x\) can also be expressed using the formula \(\mathrm{arccosh}\,x = \ln\left(x+\sqrt{x^2-1}\right)\). So: \[ \int \dfrac{4}{\sqrt{x^2-1}}dx = 4\ln\left(x+\sqrt{x^2-1}\right) + C \]

    las 

  19. \(\displaystyle\int \dfrac{x^2}{1-x^2} dx\)

    Use the substitution \(x = \mathrm{tanh}\,\lambda\).

    \( \displaystyle \int \dfrac{x^2}{1-x^2} dx = \mathrm{arctanh}\,x - x + C = \dfrac{1}{2}\ln\left(\dfrac{1+x}{1-x}\right) - x + C \)

    Let \(x=\tanh\,\lambda\). Then \(dx=\mathrm{sech}^2\,\lambda\,\tanh\,\lambda\,d\lambda\). \[\begin{aligned} \int \dfrac{x^2}{1-x^2} \,dx &= \int\dfrac{\tanh^2\lambda}{1-\tanh^2\lambda}\, \mathrm{sech}^2\lambda\,d\lambda \\ &= \int \dfrac{\tanh^2\lambda}{\mathrm{sech}^2\,\lambda}\, \mathrm{sech}^2\lambda\,d\lambda \\ &= \int \tanh^2\lambda \,d\lambda \\ \end{aligned}\] Next, we use the identity \(\tanh^2 t = 1 - \mathrm{sech}^2\,t\). \[\begin{aligned} \int \dfrac{x^2}{1-x^2} dx &= \int 1 - \mathrm{sech}^2\,\lambda \,d\lambda\\ &= \lambda - \tanh\lambda + C \end{aligned}\] Finally, we substitute back for \(x\). Since \(x = \tanh\lambda\), we have \(\lambda = \mathrm{arctanh}\,x\). So: \[ \int \dfrac{x^2}{1-x^2} dx = \mathrm{arctanh}\,x - x + C \] While this is a fine answer, \(\mathrm{arctanh}\,x\) can also be expressed using the formula \(\mathrm{arctanh}\,x = \dfrac{1}{2}\ln\left(\dfrac{1+x}{1-x}\right)\). \[ \int \dfrac{x^2}{1-x^2} dx = \dfrac{1}{2}\ln\left(\dfrac{1+x}{1-x}\right) - x + C \]

    las 

    This integral can also be done by subtracting and adding \(1\) from the numerator. \[\begin{aligned} \int \dfrac{x^2}{1-x^2} \,dx &=\int \dfrac{x^2-1+1}{1-x^2} \,dx \\ &=\int -1+\dfrac{1}{1-x^2} \,dx \\ &=-x+\mathrm{arctanh}\,x+C \end{aligned}\]

  20. \(\displaystyle\int \dfrac{2x-12}{\sqrt{x^2+1}} dx\)

    Use the substituation \(x = \sinh \lambda \)

    \(\begin{aligned} \int \dfrac{2x-12}{\sqrt{x^2+1}} dx &=2\sqrt{1 + x^2}-12\,\mathrm{arcsinh}\,x + C \\ &=2\sqrt{1 + x^2}-12 \ln(x + \sqrt{x^2+1})+C \end{aligned}\)

    We let \(x = \sinh \lambda\). Then \(dx = \cosh \lambda\,d\lambda\). \[\begin{aligned} \int \dfrac{2x-12}{\sqrt{x^2+1}} dx &= \int \dfrac{2\sinh\lambda-12}{\sqrt{\sinh^2\lambda+1}}\cosh\lambda \,d\lambda \\ &= \int (2\sinh\lambda-12) \,d\lambda \\ &= 2\cosh\lambda-12\lambda + C \\ \end{aligned}\] Finally, we use the identity \(\cosh\lambda = \sqrt{1+\sinh^2\lambda} = \sqrt{1+x^2}\) and \(\lambda = \mathrm{arcsinh}\,x\) to express the integral in terms of \(x\): \[ \int \dfrac{2x-12}{\sqrt{x^2+1}} dx = 2\sqrt{1 + x^2}-12\,\mathrm{arcsinh}\,x + C \] Although this is a fine answer, we can also write the answer without inverse hyperbolic functions. We know that \(\mathrm{arcsinh}\,x = \ln(x + \sqrt{x^2+1})\), thus: \[ \int \dfrac{2x-12}{\sqrt{x^2+1}} dx = 2\sqrt{1 + x^2}-12 \ln(x + \sqrt{x^2+1})+C \]

    las 

  21. \(\displaystyle\int \dfrac{\sqrt{4x^2-9}}{x^2} \,dx\)

    Use the substitution \(x = a\cosh\lambda\) with some constant \(a\).

    \(\begin{aligned} \int \dfrac{\sqrt{4x^2-9}}{x^2} \,dx &=2\,\mathrm{arccosh}\,\dfrac{2x}{3} - 2\sqrt{1-\dfrac{9}{4x^2}} \\ &=2\ln\left(2x + \sqrt{4x^2-9}\right) - \dfrac{\sqrt{4x^2-9}}{x} + C \end{aligned}\)

    Let \(4x^2=9\cosh^2\lambda\). Then \(x = \dfrac{3}{2}\cosh\lambda\) and thus, \(dx = \dfrac{3}{2}\sinh\lambda\,d\lambda\). Then: \[\begin{aligned} \int \dfrac{\sqrt{4x^2-9}}{x^2} &= \int \dfrac{\,\sqrt{9\cosh^2\lambda-9}\,}{\dfrac94\cosh^2\lambda} \,\dfrac{3}{2} \sinh\lambda\,d\lambda \\ &=2\int \dfrac{\sinh^2\lambda}{\cosh^2\lambda}\,d\lambda\\ &=2\int \mathrm{tanh}^2\,\lambda\,d\lambda \\ &=2\int (1 - \mathrm{sech}^2\,\lambda)\,d\lambda \\ &= 2\lambda - 2\mathrm{tanh}\,\lambda + C \end{aligned}\] Since \(x = \dfrac{3}{2}\cosh\lambda\), we have \(\lambda = \mathrm{arccosh}\,\dfrac{2x}{3}\) and \[\begin{aligned} \tanh \lambda &=\dfrac{\sinh\lambda}{\cosh\lambda} =\dfrac{\sqrt{\cosh^2\lambda-1}}{\cosh\lambda} \\ &=\dfrac{\sqrt{\dfrac{4x^2}{9}-1}}{\dfrac{2x}{3}} =\dfrac{\sqrt{4x^2-9}}{2x} \end{aligned}\] So the integral becomes: \[ \int \dfrac{\sqrt{4x^2-9}}{x^2} =2\,\mathrm{arccosh}\,\dfrac{2x}{3} - \dfrac{\sqrt{4x^2-9}}{x} + C \] Although this is a perfectly fine answer, we can also rewrite it without hyperbolic functions. Since \(\mathrm{arccosh}\,x =\ln\left(x + \sqrt{x^2-1}\right)\), we conclude \(\mathrm{arccosh}\,\dfrac{2x}{3} =\ln\left(\dfrac{2x}{3} + \sqrt{\dfrac{4x^2}{9}-1}\right)\). So: \[\begin{aligned} \int \dfrac{\sqrt{4x^2-9}}{x^2} &= 2\ln\left(\dfrac{2x}{3} + \dfrac{\sqrt{4x^2-9}}{3}\right) - \dfrac{\sqrt{4x^2-9}}{x} + C \\ &= 2\ln\left(2x + \sqrt{4x^2-9}\right) + 2\ln\left(\dfrac{1}{3}\right) - \dfrac{\sqrt{4x^2-9}}{x} + C \\ &= 2\ln\left(2x + \sqrt{4x^2-9}\right) - \dfrac{\sqrt{4x^2-9}}{x} + C' \end{aligned}\] where we have absorbed the \(2\ln\left(\dfrac{1}{3}\right)\) into the constant \(C'\). This simplification will be useful in the remark.

    las 

    This integral can also be solved using an ordinary trig substitutions. In particular, let \(4x^2 =9\sec^2\theta\). Then, \(x = \dfrac{3}{2}\sec\theta\) and \(dx = \dfrac{3}{2}\sec\theta\tan\theta\,d\theta\). So: \[\begin{aligned} \int \dfrac{\sqrt{4x^2-9}}{x^2} &=\int \dfrac{\,\sqrt{9\sec^2\theta-9}\,}{\dfrac{9}{4}\sec^2\theta} \,\dfrac{3}{2}\sec\theta\tan\theta\,d\theta \\ &= 2\int \dfrac{\tan^2\theta}{\sec\theta} \,d\theta =2\int \dfrac{\sec^2\theta - 1}{\sec\theta} \,d\theta \\ &=2\int \sec\theta - \cos\theta d\theta \\ &= 2\ln|\sec\theta + \tan\theta| - 2\sin\theta + C \end{aligned}\] To express this in terms of \(x\), since \(\sec\theta=\dfrac{2x}{3}\), we draw a triangle with hypotenuse \(2x\), adjacent side \(3\), and opposite side \(\sqrt{4x^2-9}\). Then: \[\begin{aligned} \int \dfrac{\sqrt{4x^2-9}}{x^2} &=2\ln\left|\dfrac{2x}{3}+\sqrt{4x^2-9}\right| - \dfrac{\sqrt{4x^2-9}}{x} + C \end{aligned}\] This is the same as one step in the previous answer.

  22. Find the area of the hyperbolic sector between the hyperbola \(x^2-y^2=1\), the \(x\)-axis and the line between the origin and the point \((\cosh\lambda,\sinh\lambda)\).

    eg_hyper_sector

    Instead of trying to calculate the area of the sector directly, find the area of the triangle and subtract the area below the curve.

    eg_hyper_sector_perp

    \(\displaystyle A_\text{sector}= \dfrac{1}{2} \lambda\)

    To find for the area of the hyperbolic sector, we will first find the area of the triangle with vertices \((0,0),(\cosh\lambda,0), (\cosh\lambda, \sinh\lambda)\) and subtract the area below the hyperbola.

    eg_hyper_sector_perp

    The area of the triangle is \[ A_\triangle=\dfrac12\,\text{base}\;\text{height} =\dfrac12 \cosh\lambda \sinh\lambda \] The area under the hyperbola is \[ A_\text{hyp}=\int_1^{\cosh\lambda} \sqrt{x^2-1}\,dx \] To integrate, we use the hyperbolic trig subtitution, \(x = \cosh t\). Then \(dx = \sinh t \,dt\). So: \[\begin{aligned} A_\text{hyp} &=\int_{x=1}^{\cosh\lambda}\sqrt{\cosh^2t - 1}\,\sinh t \,dt \\ &=\int_{x=1}^{\cosh\lambda} \sinh^2t \,dt \end{aligned}\] Next, we substitute \(\sinh^2t=\dfrac{1}{2}(\cosh{2t}-1)\): \[\begin{aligned} A_\text{hyp} &= \int_{x=1}^{\cosh\lambda}\dfrac{1}{2}( \cosh{2t}-1)\,dt \\ &= \dfrac{1}{2}\left[\dfrac{1}{2}\sinh{2t}-t\right]_{x=1}^{\cosh\lambda} \end{aligned}\] Before substituting back, we use the identity \(\sinh{2t}=2\cosh t\sinh t\): \[\begin{aligned} A_\text{hyp} &=\dfrac{1}{2}\left[\rule{0pt}{10pt}\cosh t\sinh t-t\right]_{x=1}^{\cosh\lambda} \\ &=\dfrac{1}{2}\left[\rule{0pt}{10pt}\cosh t\sqrt{\cosh^2 t-1}-t\right]_{x=1}^{\cosh\lambda} \end{aligned}\] To substitute back to \(x\) values, we use our definition \(\cosh t=x\) and also \(t = \mathrm{arccosh}\,x\), as well as \(\mathrm{arccosh}\,(1)=0\): \[\begin{aligned} A_\text{hyp} &=\dfrac{1}{2}\left[x\sqrt{x^2-1}-\mathrm{arccosh}\,x\right]_1^{\cosh\lambda} \\ &=\dfrac{1}{2}\cosh\lambda \sqrt{\cosh^2\lambda-1}-\dfrac{1}{2}\mathrm{arccosh}(\cosh\lambda)-0 \\ &=\dfrac{1}{2}\cosh\lambda \sinh\lambda-\dfrac{1}{2}\lambda \end{aligned}\] Finally, we combine this with the area of the triangle to get: \[\begin{aligned} A_\text{sector}&=A_\triangle-A_\text{hyp} \\ &= \dfrac{1}{2}\cosh\lambda \sinh\lambda - \left[ \dfrac{1}{2}\cosh\lambda \sinh\lambda-\dfrac{1}{2}\lambda\right] \\ &= \dfrac{1}{2}\lambda \end{aligned}\]

    las 

    It's interesting compare the area of a hyperbolic sector to the area of a circlar sector. The full unit circle has area \(A_\bigcirc=\pi r^2=\pi\). The fraction inside the sector is \(\dfrac{\theta}{2\pi}\) where \(\theta\) is measured in radians. So the area of the sector is: \[ A_\measuredangle=\dfrac{\theta}{2\pi}\pi=\dfrac{1}{2}\theta \]

    eg_circle_sector

    This relationship is the same as the hyperbolic area, \(A = \dfrac{1}{2}\lambda\).

  23. Review Exercises

    Evaluate using any appropriate method.

  24. \(\displaystyle \int \dfrac{\sqrt{x^{2}-9}}{x}\,dx\)

    Let \(x=3\sec(\theta)\).

    \( \displaystyle \int \dfrac{\sqrt{x^{2}-9}}{x}\,dx=\sqrt{x^{2}-9}-3\text{arcsec}\dfrac{x}{3} +C\)

    Let \(x=3\sec\theta\). Then \(dx=3\sec{\theta}\tan{\theta}\,d\theta\). \[\begin{aligned} \int \dfrac{\sqrt{x^{2}-9}}{x}\,dx &=\int \dfrac{\sqrt{9\sec^{2}{\theta}-9}}{3\sec\theta}\,3\sec\theta\tan\theta\,d\theta \\ &=\int 3\tan^2{\theta}\,d\theta \\ &=\int 3\sec^2{\theta}-3\,d\theta \\ &=3\tan{\theta}-3\,\theta+C \\ &=3\sqrt{\sec^2{\theta}-1}-3\,\text{arcsec}{\dfrac{x}{3}}+C \\ &=\sqrt{x^2-9}-3\,\text{arcsec}{\dfrac{x}{3}}+C \end{aligned}\]

    pm 

    \[\begin{aligned} \dfrac{d}{dx}\left(\sqrt{x^{2}-9}-3\text{arcsec}\dfrac{x}{3}\right) &=\dfrac{2x}{2\sqrt{x^2-9}}-3{\dfrac{\dfrac{1}{3}}{\left(\dfrac{x}{3}\right){\sqrt{\left(\dfrac{x}{3}\right)^{2}-1}}}} \\ &=\dfrac{x}{\sqrt{x^2-9}}-\dfrac{9}{x{\sqrt{x^2-9}}} \\ &=\dfrac{x^2-9}{x\sqrt{x^2-9}} \\ &=\dfrac{\sqrt{x^2-9}}{x} \end{aligned}\]

  25. \(\displaystyle \int \dfrac{dx}{\sqrt{x^{2}+2x}}\)

    Complete the square and then let \(x+1=\sec(\theta)\).

    \( \displaystyle \int \dfrac{dx}{\sqrt{x^{2}+2x}} =\ln\left|1+x+\sqrt{\left(x^{2}+2x\right)}\right|+C\)

    We first complete the square: \[ x^2+2x=(x+1)^2-1 \] So we let \(x+1=\sec{\theta}\). Then \(dx=\tan{\theta}\sec{\theta}\,d\theta\). \[\begin{aligned} \int \dfrac{dx}{\sqrt{x^2+2x}} &=\int \dfrac{dx}{\sqrt{(x+1)^2-1}} \\ &=\int \dfrac{\tan{\theta}\sec{\theta}}{\tan{\theta}}\,d\theta \\ &=\int \sec{\theta}\,d\theta \\ &=\ln|\sec{\theta}+\tan{\theta}|+C \\ &=\ln|x+1+\sqrt{\sec^2{\theta}-1}|+C \\ &=\ln|x+1+\sqrt{(x+1)^2-1}|+C \\ &=\ln|x+1+\sqrt{x^2+2x}|+C \end{aligned}\]

    pm 

    \[\begin{aligned} \dfrac{d}{dx}\ln|x+1+\sqrt{x^2+2x}| &=\dfrac{1+\dfrac{2x+2}{2\sqrt{x^2+2x}}}{x+1+\sqrt{x^2+2x}} \\ &=\dfrac{2\sqrt{x^2+2x}+2x+2}{2x\sqrt{x^2+2x}+2\sqrt{x^2+2x}+2x^2+4x} \\ &=\dfrac{\sqrt{x^2+2x}+x+1}{x\sqrt{x^2+2x}+\sqrt{x^2+2x}+x^2+2x} \\ &=\dfrac{x+1+\sqrt{x^2+2x}}{\sqrt{x^2+2x}(x+1+\sqrt{x^2+2x})} \\ &=\dfrac{1}{\sqrt{x^2+2x}} \end{aligned}\]

  26. \(\displaystyle \int \dfrac{x}{\sqrt{9-x^{2}}}\,dx\)

    Let \(x=3\sin \theta\).

    \( \displaystyle \int \dfrac{x}{\sqrt{9-x^{2}}}\,dx=-\sqrt{9-x^{2}}+C\)

    Let \(x=3\sin \theta\). Then \(dx=3\cos\theta\) and: \[\begin{aligned} \int \dfrac{x}{\sqrt{9-x^2}} \,dx &= \int \dfrac{3\sin\theta}{3\cos\theta}3\cos\theta\,d\theta \\ &= \int 3\sin\theta\,d\theta \\ &=-3\cos(\theta)+C \\ &=-\sqrt{9-9 \sin^2 \theta} +C \\ &=-\sqrt{9-x^2}+C \end{aligned}\]

    pm 

    \[\begin{aligned} \dfrac{d}{dx}\left(-\sqrt{9-x^2}\right) &=-\dfrac{-2x}{2\sqrt{9-x^2}} \\ &=\dfrac{x}{\sqrt{9-x^2}} \end{aligned}\]

    This can also be done by the ordinary substitution \(u=9-x^2\). Then \(du=-2x\,dx\) and so: \[\begin{aligned} \int \dfrac{x}{\sqrt{9-x^{2}}}\;dx &=\int \dfrac{x}{\sqrt{u}}\cdot\dfrac{1}{-2x}\;du =-\int \dfrac{1}{2\sqrt{u}}\;du \\ &=-u^{1/2}+C =-\sqrt{9-x^2}+C\\ \end{aligned}\]

  27. \(\displaystyle \int \dfrac{dx}{x^{2}\sqrt{9-x^{2}}}\)

    Recall \(\dfrac{d}{dx}\cot x=-\csc ^{2}x\).

    \( \displaystyle \int \dfrac{dx}{x^{2}\sqrt{9-x^{2}}}=-\,\dfrac{\sqrt{9-x^2}}{9x}+C\)

    Let \(x=3\sin(\theta)\). Then \(dx=3\cos(\theta)d\theta\). \[\begin{aligned} \int \dfrac{1}{x^2\sqrt{9-x^2}} dx &=\int \dfrac{1}{9\sin^2(\theta)\sqrt{9-9\sin^2(\theta)}} 3\cos\theta\,d\theta \\ &=\int \dfrac{3\cos\theta}{9\sin^2(\theta)3\cos\theta} \,d\theta \\ &=\dfrac{1}{9}\int \csc^2(\theta)\,d\theta =-\,\dfrac{1}{9}\cot(\theta)+C \\ &=-\,\dfrac{1}{9}\dfrac{\sqrt{1-\sin^2(\theta)}}{\sin(\theta)}+C\\ &=-\,\dfrac{1}{9}\dfrac{\;\sqrt{1-\dfrac{x^2}{9}}\;}{\dfrac{x}{3}}+C\\ &=-\,\dfrac{\sqrt{9-x^2}}{9x}+C \end{aligned}\]

    pm 

    \[\begin{aligned} \dfrac{d}{dx}\left(-\,\dfrac{\sqrt{9-x^2}}{9x}\right) &=-\,\dfrac{9x(\sqrt{9-x^2})'-\sqrt{9-x^2}9}{81x^2} \\ &=-\,\dfrac{9x\dfrac{-2x}{2\sqrt{9-x^2}}-9\sqrt{9-x^2}}{81x^2} \\ &=\dfrac{9x\dfrac{x}{\sqrt{9-x^2}}+9\sqrt{9-x^2}}{81x^2} \\ &=\dfrac{\dfrac{9x^2}{\sqrt{9-x^2}}+\dfrac{9(9-x^2)}{\sqrt{9-x^2}}}{81x^2} \\ &=\dfrac{81}{81x^2\sqrt{9-x^2}} \\ &=\dfrac{1}{x^2\sqrt{9-x^2}} \end{aligned}\]

  28. \(\displaystyle \int \dfrac{x^{2}}{\sqrt{1-x^{2}}}\,dx \)

    Recall the integral of \(\sin^2(x)\) is \(\dfrac{x-\sin(x)\cos(x)}{2}\).

    \( \displaystyle \int \dfrac{x^{2}}{\sqrt{1-x^{2}}}\,dx=-\dfrac{1}{2}x\sqrt{1-x^{2}}+\dfrac{1 }{2}\arcsin x+C \)

    Let \(x=\sin{\theta}\). Then \(dx=\cos{\theta}\,d\theta\). \[\begin{aligned} \int \dfrac{x^{2}}{\sqrt{1-x^{2}}}\,dx &=\int \dfrac{\sin^2{u}}{\cos{u}} \cos{u}\;du \\ &=\int \sin^2{u}\;du \\ &=\dfrac{u-\sin(u)\cos(u)}{2}+C \\ &=-\dfrac{1}{2}x\sqrt{1-x^{2}}+\dfrac{1}{2}\arcsin x+C \end{aligned}\]

    \[\begin{aligned} \dfrac{d}{dx}&\left(-\dfrac{1}{2}x\sqrt{1-x^{2}}+\dfrac{1}{2}\arcsin{x}\right) \\ &=-\dfrac{\sqrt{1-x^2}}{2}-\dfrac{x}{2}\cdot \dfrac{-2x}{2\sqrt{1-x^2}}+\dfrac{1}{2\sqrt{1-x^2}} \\ &=\dfrac{x^2-1}{2\sqrt{1-x^2}}+\dfrac{x^2}{2\sqrt{1-x^2}}+\dfrac{1}{2\sqrt{1-x^2}} \\ &=\dfrac{x^2}{\sqrt{1-x^2}} \end{aligned}\]


  29. Find the area of the surface generated by revolving the graph of \( y=\sqrt{x^{2}+2}\) between \(x=0\) and \(x=1\) about the \(x\)-axis.

    It's OK to use the antiderivative of \(\sec^3 x\): \[ \int \sec^3 \theta\,d\theta =\dfrac{1}{2}\sec\theta\tan\theta+\dfrac{1}{2}\ln|\sec\theta+\tan\theta|+C \] which was derived in the chapter on trig integrals.

    \(\displaystyle A=\int_{0}^{1} 2\pi \sqrt{x^{2}+2}\sqrt{1+\left( \dfrac{x}{\sqrt{x^{2}+2}} \right) ^{2}}dx\) \(=2\pi +\sqrt{2}\pi \ln \left( \sqrt{2}+1\right) \)

    Since we are revolving about the \(x\)-axis, the radius is \(r=y=\sqrt{x^{2}+2}\). The differential of arclength is \(ds=\sqrt{1+\left(\dfrac{dy}{dx}\right)^2} dx\). So the surface area is: \[\begin{aligned} A&=\int_0^1 2\pi y \; ds \\ &=\int_0^1 2\pi \sqrt{x^2+2} \sqrt{1+\left(\dfrac{dy}{dx}\right)^2} dx \\ &= \int_0^1 2\pi \sqrt{x^2+2} \sqrt{1+\left(\dfrac{x}{\sqrt{x^2+2}}\right)^2} dx \\ &= \int_0^1 2\pi \sqrt{x^2+2} \sqrt{\dfrac{x^2+2}{x^2+2}+\dfrac{x^2}{x^2+2}} \; dx \\ &= 2\sqrt{2} \pi \int_0^1 \sqrt{x^2+1} \; dx \end{aligned}\] We now substitute \(x=\tan(\theta)\) and \(dx=\sec^2(\theta)\,d\theta\). Then we use the integral of \(\sec^3\theta\). (See the hint.) \[\begin{aligned} A&= 2\sqrt{2} \pi \int_0^1 \sqrt{x^2+1}\,dx = 2\sqrt{2} \pi \int_0^{\pi/4} \sec^3(\theta)\,d\theta \\ &=2\sqrt{2}\pi\left[\dfrac{1}{2}\sec(\theta)\tan(\theta)+\dfrac{1}{2}\ln|\sec(\theta)+\tan(\theta)|\right]_0^{\pi/4} \\ &=\sqrt{2}\pi\left[\sec\left(\dfrac{\pi}{4}\right)\tan\left(\dfrac{\pi}{4}\right) +\ln\left|\sec\left(\dfrac{\pi}{4}\right)+\tan\left(\dfrac{\pi}{4}\right)\right|\right. \\ &\qquad\qquad\left.-\sec(0)\tan(0)-\ln\left|\sec(0)+\tan(0)\right|\rule{0pt}{10pt}\right]\\ &=2\sqrt{2}\pi\left[\dfrac{\sqrt{2}}{2}+\dfrac{1}{2}\ln\left(\sqrt{2}+1\right)-\dfrac{1}{2}\ln(1)\right] \\ &=2\pi+\sqrt{2}\pi\ln\left(\sqrt{2}+1\right) \end{aligned}\]

  30. PY: All Checked.

© MYMathApps

Supported in part by NSF Grant #1123255